2021 AMC 12A Problems/Problem 11

Problem

A laser is placed at the point $(3,5)$. The laser beam travels in a straight line. Larry wants the beam to hit and bounce off the $y$-axis, then hit and bounce off the $x$-axis, then hit the point $(7,5)$. What is the total distance the beam will travel along this path?

$\textbf{(A) }2\sqrt{10} \qquad \textbf{(B) }5\sqrt2 \qquad \textbf{(C) }10\sqrt2 \qquad \textbf{(D) }15\sqrt2 \qquad \textbf{(E) }10\sqrt5$

Diagram

[asy] /* Made by MRENTHUSIASM */ size(200);   int xMin = -3; int xMax = 9; int yMin = -3; int yMax = 7;  draw((xMin,0)--(xMax,0),black+linewidth(1.5),EndArrow(5)); draw((0,yMin)--(0,yMax),black+linewidth(1.5),EndArrow(5)); label("$x$",(xMax,0),(2,0)); label("$y$",(0,yMax),(0,2));  pair A = (3,5); pair B = (0,2); pair C = (2,0); pair D = (7,5);  draw(A--B--C--D,red); dot(A,linewidth(3.5)); dot(B,linewidth(3.5)); dot(C,linewidth(3.5)); dot(D,linewidth(3.5)); label("$(3,5)$",A,(0,2)); label("$(7,5)$",D,(0,2)); [/asy] ~MRENTHUSIASM

Solution 1 (Reflections)

Let $A=(3,5)$ and $D=(7,5).$ Suppose that the beam hits and bounces off the $y$-axis at $B,$ then hits and bounces off the $x$-axis at $C.$

When the beam hits and bounces off a coordinate axis, the angle of incidence and the angle of reflection are congruent. Therefore, we straighten up the path of the beam by reflections:

  1. We reflect $\overline{BC}$ about the $y$-axis to get $\overline{BC'}.$
  2. We reflect $\overline{CD}$ about the $x$-axis to get $\overline{C'D'}$ with $D'=(7,-5),$ then reflect $\overline{C'D'}$ about the $y$-axis to get $\overline{C'D''}$ with $D''=(-7,-5).$

We obtain the following diagram: [asy] /* Made by MRENTHUSIASM */ size(225);   int xMin = -9; int xMax = 9; int yMin = -7; int yMax = 7;  draw((xMin,0)--(xMax,0),black+linewidth(1.5),EndArrow(5)); draw((0,yMin)--(0,yMax),black+linewidth(1.5),EndArrow(5)); label("$x$",(xMax,0),(2,0)); label("$y$",(0,yMax),(0,2));  pair A = (3,5); pair B = (0,2); pair C = (2,0); pair D = (7,5); pair E = (-2,0); pair F = (7,-5); pair G = (-7,-5);  draw(A--B--C--D,red); draw(B--E,heavygreen+dashed); draw(C--F,heavygreen+dashed); draw(E--G,heavygreen+dashed); dot("$A(3,5)$",A,(0,2),linewidth(3.5)); dot("$B$",B,(-2,0),linewidth(3.5)); dot("$C$",C,(0,-2),linewidth(3.5)); dot("$D(7,5)$",D,(0,2),linewidth(3.5)); dot("$C'$",E,(0,-2),linewidth(3.5)); dot("$D'(7,-5)$",F,(0,-2),linewidth(3.5)); dot("$D''(-7,-5)$",G,(0,-2),linewidth(3.5)); [/asy] The total distance that the beam will travel is \begin{align*} AB+BC+CD&=AB+BC+CD' \\ &=AB+BC'+C'D'' \\ &=AD'' \\ &=\sqrt{((3-(-7))^2+(5-(-5))^2} \\ &=\boxed{\textbf{(C) }10\sqrt2}. \end{align*} ~MRENTHUSIASM (Solution)

~JHawk0224 (Proposal)

Solution 2 (Parallelogram)

Define points $A,B,C,$ and $D$ as Solution 1 does. Moreover, let $E$ be a point on $\overline{CD}$ such that $\overline{BE}$ is perpendicular to the $y$-axis, and $F$ be a point on $\overline{BE}$ such that $\overline{CF}$ is perpendicular to the $x$-axis, as shown below. [asy] /* Made by MRENTHUSIASM */ size(200);   int xMin = -3; int xMax = 9; int yMin = -3; int yMax = 7;  draw((xMin,0)--(xMax,0),black+linewidth(1.5),EndArrow(5)); draw((0,yMin)--(0,yMax),black+linewidth(1.5),EndArrow(5)); label("$x$",(xMax,0),(2,0)); label("$y$",(0,yMax),(0,2));  pair A = (3,5); pair B = (0,2); pair C = (2,0); pair D = (7,5); pair E = (4,2); pair F = (2,2);  draw(A--B--C--D,red); draw(A--D^^B--E^^C--F,heavygreen+dashed); dot("$A(3,5)$",A,(0,2),linewidth(3.5)); dot("$B$",B,(-2,0),linewidth(3.5)); dot("$C$",C,(0,-2),linewidth(3.5)); dot("$D(7,5)$",D,(0,2),linewidth(3.5)); dot("$E$",E,(2,0),linewidth(3.5)); dot("$F$",F,(0,1.5),linewidth(3.5)); [/asy] When the beam hits and bounces off a coordinate axis, the angle of incidence and the angle of reflection are congruent, from which $\angle ABF=\angle CBF$ and $\angle BCF=\angle ECF.$ We conclude that $\triangle BCF\cong\triangle ECF$ by ASA, so $\angle CBF=\angle CEF.$ It follows that $\angle ABF=\angle CEF$ by transitive, so $\overline{AB}\parallel\overline{CD}$ by the Converse of the Alternate Interior Angles Theorem.

Note that $\overline{AD}\parallel\overline{BE}.$ Since the opposite sides are parallel, quadrilateral $ABED$ is a parallelogram. From $\triangle BCF\cong\triangle ECF,$ we get $BF=EF=2,$ so $C=(2,0).$

Let $B=(0,b).$ We equate the slopes of $\overline{AB}$ and $\overline{DC}:$ \[\frac{5-b}{3-0}=\frac{5-0}{7-2},\] from which $b=2,$ or $B=(0,2).$

By the Distance Formula, we have $AB=3\sqrt2,BC=2\sqrt2,$ and $CD=5\sqrt2.$ The total distance that the beam will travel is \[AB+BC+CD=\boxed{\textbf{(C) }10\sqrt2}.\]

Remark

When a straight line hits and bounces off a coordinate axis at point $P,$ the ray entering $P$ and the ray leaving $P$ always have negative slopes. In this problem, $\overline{AB}$ and $\overline{BC}$ have negative slopes; $\overline{BC}$ and $\overline{CD}$ have negative slopes. So, $\overline{AB}$ and $\overline{CD}$ have the same slope, or $\overline{AB}\parallel\overline{CD}.$

~MRENTHUSIASM

Solution 3 (Educated Guess)

Define points $A,B,C,$ and $D$ as Solution 1 does.

Since choices $\textbf{(B)}, \textbf{(C)},$ and $\textbf{(D)}$ all involve $\sqrt2,$ we suspect that one of them is the correct answer. We take a guess in faith that $\overline{AB},\overline{BC},$ and $\overline{CD}$ all form $45^\circ$ angles with the coordinate axes, from which $B=(0,2)$ and $C=(2,0).$ The given condition $D=(7,5)$ verifies our guess, as shown below. [asy] /* Made by MRENTHUSIASM */ size(200);   int xMin = -3; int xMax = 9; int yMin = -3; int yMax = 7;  //Draws the horizontal gridlines void horizontalLines() {   for (int i = yMin+1; i < yMax; ++i)   {     draw((xMin,i)--(xMax,i), mediumgray+linewidth(0.4));   } }  //Draws the vertical gridlines void verticalLines() {   for (int i = xMin+1; i < xMax; ++i)   {     draw((i,yMin)--(i,yMax), mediumgray+linewidth(0.4));   } }  //Draws the horizontal ticks void horizontalTicks() {   for (int i = yMin+1; i < yMax; ++i)   {     draw((-3/16,i)--(3/16,i), black+linewidth(1));   } }  //Draws the vertical ticks void verticalTicks() {   for (int i = xMin+1; i < xMax; ++i)   {     draw((i,-3/16)--(i,3/16), black+linewidth(1));   } }  horizontalLines(); verticalLines(); horizontalTicks(); verticalTicks(); draw((xMin,0)--(xMax,0),black+linewidth(1.5),EndArrow(5)); draw((0,yMin)--(0,yMax),black+linewidth(1.5),EndArrow(5)); label("$x$",(xMax,0),(2,0)); label("$y$",(0,yMax),(0,2));  pair A = (3,5); pair B = (0,2); pair C = (2,0); pair D = (7,5);  draw(A--B--C--D,red); dot(A,linewidth(3.5)); dot(B,linewidth(3.5)); dot(C,linewidth(3.5)); dot(D,linewidth(3.5)); label("$A(3,5)$",A,(0,2),UnFill); label("$B$",B,(-2,0),UnFill); label("$C$",C,(0,-2),UnFill); label("$D(7,5)$",D,(0,2),UnFill); [/asy] Following the last paragraph of Solution 2 gives the answer $\boxed{\textbf{(C) }10\sqrt2}.$

~MRENTHUSIASM

Solution 4 (System of Linear Equations)

Denote $(3, 5)$ as point $P$ and $(7, 5)$ as point $S$. Define the point $Q$ on the $y$-axis that the laser hits as $(0, y_1)$ and the point $R$ on the $x$-axis that the laser hits as $(x_1,0)$. The laser will bounce off of the two axes at right angles, so we have that line $PQ \parallel RS$, meaning they have the same slope which we will denote as $k$. Line $QR$ will be perpendicular to both $PQ$ and $RS$, meaning it will have slope $-\frac{1}{k}$.

We can write the equations for lines $PQ$ and $RS$ in point-slope form, and then plug in points $Q$ and $R$ into those respective equations to get equations in terms of our three variables. We can also write the equation for line $QR$ in slope-intercept form and plug in point $R$. Doing this yields \begin{align*}     y_1-5&=k(0-3)\\     0-5&=k(x_1-7)\\     0&=-\frac{1}{k}x_1+y_1. \end{align*} We can simplify this into the following cubic equation in terms of $k$: \[-3k^3+5k^2-7k+5=0.\] Luckily, we notice that the coefficients sum up to zero, so $k=1$. Now we can solve for the other two, yielding $x_1=y_1=2$. Now we simply have three $45^{\circ}$-$45^{\circ}$-$90^{\circ}$ triangles with leg lengths of $3, 2,$ and $5$, so our final distance is \[3\sqrt{2}+2\sqrt{2}+5\sqrt{2}=\boxed{\textbf{(C) }10\sqrt2}.\] ~Mooshiros

Video Solution by OmegaLearn (Using Reflections and Distance Formula)

https://youtu.be/e7tNtd-fgeo

~ pi_is_3.14

Video Solution by Hawk Math

https://www.youtube.com/watch?v=AjQARBvdZ20

Video Solution by TheBeautyofMath

https://youtu.be/ySWSHyY9TwI

~IceMatrix

Video Solution (Quick and Easy)

https://youtu.be/Hw-mYGUvflQ

~Education, the Study of Everything

See also

2021 AMC 12A (ProblemsAnswer KeyResources)
Preceded by
Problem 10
Followed by
Problem 12
1 2 3 4 5 6 7 8 9 10 11 12 13 14 15 16 17 18 19 20 21 22 23 24 25
All AMC 12 Problems and Solutions

The problems on this page are copyrighted by the Mathematical Association of America's American Mathematics Competitions. AMC logo.png